题库> 逻辑 CR> CR-a24cck

题目选项分析

The proposal to hire ten new police officers in Middletown is quite foolish. There is sufficient funding to pay the salaries of the new officers, but not the salaries of additional court and prison employees to process the increased caseload of arrests and convictions that new officers usually generate.

>Which of the following, if true, will most seriously weaken the conclusion drawn above?

【选项】When one major city increased its police force by 19% last year, there were 40% more arrests and 13% more convictions

选项是否正确?

答对用户笔记

题目基本信息

  • 所属科目:逻辑CR
  • 题目来源1:PREP07 Test 2-58
难度: 难